An analogue to Euler's sum of powers conjecture












4












$begingroup$


What if instead of considering $$a_1^3+a_2^3+a_3^3=b_1^3 \
a_1^4+a_2^4+a_3^4+a_4^4=b_1^4 \
vdots $$

we instead considered
$$a_1^3+a_2^3+a_3^3=b_1^3+b_2^3 \
a_1^4+a_2^4+a_3^4+a_4^4=b_1^4+b_2^4+b_3^4 \
a_1^5+a_2^5+a_3^5+a_4^5+a_5^5=b_1^5+b_2^5+b_3^5+b_4^5 \
vdots $$
My gut feeling is that this will have infinitely many solutions.



Note: $a_i,b_i$ are all positive integers.










share|cite|improve this question











$endgroup$

















    4












    $begingroup$


    What if instead of considering $$a_1^3+a_2^3+a_3^3=b_1^3 \
    a_1^4+a_2^4+a_3^4+a_4^4=b_1^4 \
    vdots $$

    we instead considered
    $$a_1^3+a_2^3+a_3^3=b_1^3+b_2^3 \
    a_1^4+a_2^4+a_3^4+a_4^4=b_1^4+b_2^4+b_3^4 \
    a_1^5+a_2^5+a_3^5+a_4^5+a_5^5=b_1^5+b_2^5+b_3^5+b_4^5 \
    vdots $$
    My gut feeling is that this will have infinitely many solutions.



    Note: $a_i,b_i$ are all positive integers.










    share|cite|improve this question











    $endgroup$















      4












      4








      4


      0



      $begingroup$


      What if instead of considering $$a_1^3+a_2^3+a_3^3=b_1^3 \
      a_1^4+a_2^4+a_3^4+a_4^4=b_1^4 \
      vdots $$

      we instead considered
      $$a_1^3+a_2^3+a_3^3=b_1^3+b_2^3 \
      a_1^4+a_2^4+a_3^4+a_4^4=b_1^4+b_2^4+b_3^4 \
      a_1^5+a_2^5+a_3^5+a_4^5+a_5^5=b_1^5+b_2^5+b_3^5+b_4^5 \
      vdots $$
      My gut feeling is that this will have infinitely many solutions.



      Note: $a_i,b_i$ are all positive integers.










      share|cite|improve this question











      $endgroup$




      What if instead of considering $$a_1^3+a_2^3+a_3^3=b_1^3 \
      a_1^4+a_2^4+a_3^4+a_4^4=b_1^4 \
      vdots $$

      we instead considered
      $$a_1^3+a_2^3+a_3^3=b_1^3+b_2^3 \
      a_1^4+a_2^4+a_3^4+a_4^4=b_1^4+b_2^4+b_3^4 \
      a_1^5+a_2^5+a_3^5+a_4^5+a_5^5=b_1^5+b_2^5+b_3^5+b_4^5 \
      vdots $$
      My gut feeling is that this will have infinitely many solutions.



      Note: $a_i,b_i$ are all positive integers.







      elementary-number-theory






      share|cite|improve this question















      share|cite|improve this question













      share|cite|improve this question




      share|cite|improve this question








      edited Jan 11 at 7:23







      Stupid Questions Inc

















      asked Jan 10 at 11:24









      Stupid Questions IncStupid Questions Inc

      7010




      7010






















          1 Answer
          1






          active

          oldest

          votes


















          2












          $begingroup$

          Here's a possible strategy to show that there are infinitely many solutions for the equation. For example, let's consider the first one. Suppose that there exists $x, y, zin mathbb{Z}$ with $zneq pm 1$ and satisfies $x^{3} + y^{3} = z^{3} + 2$. Then this gives one parametrization of the solution as
          $$
          (z^{m})^{3} + (z^{m})^{3} + (z^{m+1})^{3} = z^{3m}(z^{3} + 2) = z^{3m}(x^{3} + y^{3}) = (xz^{m})^{3} + (yz^{m})^{3}
          $$

          for $mgeq 0$. Also, we have a solution for the previous equation as $7^{3} + (-5)^{3} = 6^{3} + 2$, so we just proved that the equation $a_{1}^{3} + a_{2}^{3} + a_{3}^{3} = b_{1}^{3} + b_{2}^{3}$ has infinitely many integer solutions.
          If you only want positive solutions, it is also possible because we have
          $$
          1214928^{3} + 3480205^{3} = 3528875^{3} + 2.
          $$

          Similarly, it is enough to show that there exists an integer $1leq t leq k-1$ such that the equation
          $$
          x_{1}^{k} + x_{2}^{k} + cdots + x_{k-1}^{k} = ty^{k} + (k-t) = t(y^{k} - 1) + k
          $$

          has an integer (or positive integer) solution with $yneq pm 1$, which will give a parametrization
          $$
          (y^{m})^{k} + cdots + (y^{m})^{k} + (y^{m+1})^{k} + cdots + (y^{m+1})^{k} = (x_{1}y^{m})^{k} + cdots + (x_{k-1}y^{m})^{k}
          $$

          where there are $(k-t)$ many $y^{m}$'s and $t$ many $y^{m+1}$'s on LHS.






          share|cite|improve this answer









          $endgroup$













          • $begingroup$
            +1 for ingenuity, although it looks challenging, for higher powers, to find equations with suitable $t$.
            $endgroup$
            – Adam Bailey
            Jan 10 at 22:43










          • $begingroup$
            Does your strategy work if we restrict everything to the naturals?
            $endgroup$
            – Stupid Questions Inc
            Jan 13 at 19:50










          • $begingroup$
            @StupidQuestionsInc Sure, if you can find the solution of the last equation with all $x_{i}>0$ and $y>1$.
            $endgroup$
            – Seewoo Lee
            Jan 13 at 22:06











          Your Answer





          StackExchange.ifUsing("editor", function () {
          return StackExchange.using("mathjaxEditing", function () {
          StackExchange.MarkdownEditor.creationCallbacks.add(function (editor, postfix) {
          StackExchange.mathjaxEditing.prepareWmdForMathJax(editor, postfix, [["$", "$"], ["\\(","\\)"]]);
          });
          });
          }, "mathjax-editing");

          StackExchange.ready(function() {
          var channelOptions = {
          tags: "".split(" "),
          id: "69"
          };
          initTagRenderer("".split(" "), "".split(" "), channelOptions);

          StackExchange.using("externalEditor", function() {
          // Have to fire editor after snippets, if snippets enabled
          if (StackExchange.settings.snippets.snippetsEnabled) {
          StackExchange.using("snippets", function() {
          createEditor();
          });
          }
          else {
          createEditor();
          }
          });

          function createEditor() {
          StackExchange.prepareEditor({
          heartbeatType: 'answer',
          autoActivateHeartbeat: false,
          convertImagesToLinks: true,
          noModals: true,
          showLowRepImageUploadWarning: true,
          reputationToPostImages: 10,
          bindNavPrevention: true,
          postfix: "",
          imageUploader: {
          brandingHtml: "Powered by u003ca class="icon-imgur-white" href="https://imgur.com/"u003eu003c/au003e",
          contentPolicyHtml: "User contributions licensed under u003ca href="https://creativecommons.org/licenses/by-sa/3.0/"u003ecc by-sa 3.0 with attribution requiredu003c/au003e u003ca href="https://stackoverflow.com/legal/content-policy"u003e(content policy)u003c/au003e",
          allowUrls: true
          },
          noCode: true, onDemand: true,
          discardSelector: ".discard-answer"
          ,immediatelyShowMarkdownHelp:true
          });


          }
          });














          draft saved

          draft discarded


















          StackExchange.ready(
          function () {
          StackExchange.openid.initPostLogin('.new-post-login', 'https%3a%2f%2fmath.stackexchange.com%2fquestions%2f3068520%2fan-analogue-to-eulers-sum-of-powers-conjecture%23new-answer', 'question_page');
          }
          );

          Post as a guest















          Required, but never shown

























          1 Answer
          1






          active

          oldest

          votes








          1 Answer
          1






          active

          oldest

          votes









          active

          oldest

          votes






          active

          oldest

          votes









          2












          $begingroup$

          Here's a possible strategy to show that there are infinitely many solutions for the equation. For example, let's consider the first one. Suppose that there exists $x, y, zin mathbb{Z}$ with $zneq pm 1$ and satisfies $x^{3} + y^{3} = z^{3} + 2$. Then this gives one parametrization of the solution as
          $$
          (z^{m})^{3} + (z^{m})^{3} + (z^{m+1})^{3} = z^{3m}(z^{3} + 2) = z^{3m}(x^{3} + y^{3}) = (xz^{m})^{3} + (yz^{m})^{3}
          $$

          for $mgeq 0$. Also, we have a solution for the previous equation as $7^{3} + (-5)^{3} = 6^{3} + 2$, so we just proved that the equation $a_{1}^{3} + a_{2}^{3} + a_{3}^{3} = b_{1}^{3} + b_{2}^{3}$ has infinitely many integer solutions.
          If you only want positive solutions, it is also possible because we have
          $$
          1214928^{3} + 3480205^{3} = 3528875^{3} + 2.
          $$

          Similarly, it is enough to show that there exists an integer $1leq t leq k-1$ such that the equation
          $$
          x_{1}^{k} + x_{2}^{k} + cdots + x_{k-1}^{k} = ty^{k} + (k-t) = t(y^{k} - 1) + k
          $$

          has an integer (or positive integer) solution with $yneq pm 1$, which will give a parametrization
          $$
          (y^{m})^{k} + cdots + (y^{m})^{k} + (y^{m+1})^{k} + cdots + (y^{m+1})^{k} = (x_{1}y^{m})^{k} + cdots + (x_{k-1}y^{m})^{k}
          $$

          where there are $(k-t)$ many $y^{m}$'s and $t$ many $y^{m+1}$'s on LHS.






          share|cite|improve this answer









          $endgroup$













          • $begingroup$
            +1 for ingenuity, although it looks challenging, for higher powers, to find equations with suitable $t$.
            $endgroup$
            – Adam Bailey
            Jan 10 at 22:43










          • $begingroup$
            Does your strategy work if we restrict everything to the naturals?
            $endgroup$
            – Stupid Questions Inc
            Jan 13 at 19:50










          • $begingroup$
            @StupidQuestionsInc Sure, if you can find the solution of the last equation with all $x_{i}>0$ and $y>1$.
            $endgroup$
            – Seewoo Lee
            Jan 13 at 22:06
















          2












          $begingroup$

          Here's a possible strategy to show that there are infinitely many solutions for the equation. For example, let's consider the first one. Suppose that there exists $x, y, zin mathbb{Z}$ with $zneq pm 1$ and satisfies $x^{3} + y^{3} = z^{3} + 2$. Then this gives one parametrization of the solution as
          $$
          (z^{m})^{3} + (z^{m})^{3} + (z^{m+1})^{3} = z^{3m}(z^{3} + 2) = z^{3m}(x^{3} + y^{3}) = (xz^{m})^{3} + (yz^{m})^{3}
          $$

          for $mgeq 0$. Also, we have a solution for the previous equation as $7^{3} + (-5)^{3} = 6^{3} + 2$, so we just proved that the equation $a_{1}^{3} + a_{2}^{3} + a_{3}^{3} = b_{1}^{3} + b_{2}^{3}$ has infinitely many integer solutions.
          If you only want positive solutions, it is also possible because we have
          $$
          1214928^{3} + 3480205^{3} = 3528875^{3} + 2.
          $$

          Similarly, it is enough to show that there exists an integer $1leq t leq k-1$ such that the equation
          $$
          x_{1}^{k} + x_{2}^{k} + cdots + x_{k-1}^{k} = ty^{k} + (k-t) = t(y^{k} - 1) + k
          $$

          has an integer (or positive integer) solution with $yneq pm 1$, which will give a parametrization
          $$
          (y^{m})^{k} + cdots + (y^{m})^{k} + (y^{m+1})^{k} + cdots + (y^{m+1})^{k} = (x_{1}y^{m})^{k} + cdots + (x_{k-1}y^{m})^{k}
          $$

          where there are $(k-t)$ many $y^{m}$'s and $t$ many $y^{m+1}$'s on LHS.






          share|cite|improve this answer









          $endgroup$













          • $begingroup$
            +1 for ingenuity, although it looks challenging, for higher powers, to find equations with suitable $t$.
            $endgroup$
            – Adam Bailey
            Jan 10 at 22:43










          • $begingroup$
            Does your strategy work if we restrict everything to the naturals?
            $endgroup$
            – Stupid Questions Inc
            Jan 13 at 19:50










          • $begingroup$
            @StupidQuestionsInc Sure, if you can find the solution of the last equation with all $x_{i}>0$ and $y>1$.
            $endgroup$
            – Seewoo Lee
            Jan 13 at 22:06














          2












          2








          2





          $begingroup$

          Here's a possible strategy to show that there are infinitely many solutions for the equation. For example, let's consider the first one. Suppose that there exists $x, y, zin mathbb{Z}$ with $zneq pm 1$ and satisfies $x^{3} + y^{3} = z^{3} + 2$. Then this gives one parametrization of the solution as
          $$
          (z^{m})^{3} + (z^{m})^{3} + (z^{m+1})^{3} = z^{3m}(z^{3} + 2) = z^{3m}(x^{3} + y^{3}) = (xz^{m})^{3} + (yz^{m})^{3}
          $$

          for $mgeq 0$. Also, we have a solution for the previous equation as $7^{3} + (-5)^{3} = 6^{3} + 2$, so we just proved that the equation $a_{1}^{3} + a_{2}^{3} + a_{3}^{3} = b_{1}^{3} + b_{2}^{3}$ has infinitely many integer solutions.
          If you only want positive solutions, it is also possible because we have
          $$
          1214928^{3} + 3480205^{3} = 3528875^{3} + 2.
          $$

          Similarly, it is enough to show that there exists an integer $1leq t leq k-1$ such that the equation
          $$
          x_{1}^{k} + x_{2}^{k} + cdots + x_{k-1}^{k} = ty^{k} + (k-t) = t(y^{k} - 1) + k
          $$

          has an integer (or positive integer) solution with $yneq pm 1$, which will give a parametrization
          $$
          (y^{m})^{k} + cdots + (y^{m})^{k} + (y^{m+1})^{k} + cdots + (y^{m+1})^{k} = (x_{1}y^{m})^{k} + cdots + (x_{k-1}y^{m})^{k}
          $$

          where there are $(k-t)$ many $y^{m}$'s and $t$ many $y^{m+1}$'s on LHS.






          share|cite|improve this answer









          $endgroup$



          Here's a possible strategy to show that there are infinitely many solutions for the equation. For example, let's consider the first one. Suppose that there exists $x, y, zin mathbb{Z}$ with $zneq pm 1$ and satisfies $x^{3} + y^{3} = z^{3} + 2$. Then this gives one parametrization of the solution as
          $$
          (z^{m})^{3} + (z^{m})^{3} + (z^{m+1})^{3} = z^{3m}(z^{3} + 2) = z^{3m}(x^{3} + y^{3}) = (xz^{m})^{3} + (yz^{m})^{3}
          $$

          for $mgeq 0$. Also, we have a solution for the previous equation as $7^{3} + (-5)^{3} = 6^{3} + 2$, so we just proved that the equation $a_{1}^{3} + a_{2}^{3} + a_{3}^{3} = b_{1}^{3} + b_{2}^{3}$ has infinitely many integer solutions.
          If you only want positive solutions, it is also possible because we have
          $$
          1214928^{3} + 3480205^{3} = 3528875^{3} + 2.
          $$

          Similarly, it is enough to show that there exists an integer $1leq t leq k-1$ such that the equation
          $$
          x_{1}^{k} + x_{2}^{k} + cdots + x_{k-1}^{k} = ty^{k} + (k-t) = t(y^{k} - 1) + k
          $$

          has an integer (or positive integer) solution with $yneq pm 1$, which will give a parametrization
          $$
          (y^{m})^{k} + cdots + (y^{m})^{k} + (y^{m+1})^{k} + cdots + (y^{m+1})^{k} = (x_{1}y^{m})^{k} + cdots + (x_{k-1}y^{m})^{k}
          $$

          where there are $(k-t)$ many $y^{m}$'s and $t$ many $y^{m+1}$'s on LHS.







          share|cite|improve this answer












          share|cite|improve this answer



          share|cite|improve this answer










          answered Jan 10 at 13:11









          Seewoo LeeSeewoo Lee

          6,616926




          6,616926












          • $begingroup$
            +1 for ingenuity, although it looks challenging, for higher powers, to find equations with suitable $t$.
            $endgroup$
            – Adam Bailey
            Jan 10 at 22:43










          • $begingroup$
            Does your strategy work if we restrict everything to the naturals?
            $endgroup$
            – Stupid Questions Inc
            Jan 13 at 19:50










          • $begingroup$
            @StupidQuestionsInc Sure, if you can find the solution of the last equation with all $x_{i}>0$ and $y>1$.
            $endgroup$
            – Seewoo Lee
            Jan 13 at 22:06


















          • $begingroup$
            +1 for ingenuity, although it looks challenging, for higher powers, to find equations with suitable $t$.
            $endgroup$
            – Adam Bailey
            Jan 10 at 22:43










          • $begingroup$
            Does your strategy work if we restrict everything to the naturals?
            $endgroup$
            – Stupid Questions Inc
            Jan 13 at 19:50










          • $begingroup$
            @StupidQuestionsInc Sure, if you can find the solution of the last equation with all $x_{i}>0$ and $y>1$.
            $endgroup$
            – Seewoo Lee
            Jan 13 at 22:06
















          $begingroup$
          +1 for ingenuity, although it looks challenging, for higher powers, to find equations with suitable $t$.
          $endgroup$
          – Adam Bailey
          Jan 10 at 22:43




          $begingroup$
          +1 for ingenuity, although it looks challenging, for higher powers, to find equations with suitable $t$.
          $endgroup$
          – Adam Bailey
          Jan 10 at 22:43












          $begingroup$
          Does your strategy work if we restrict everything to the naturals?
          $endgroup$
          – Stupid Questions Inc
          Jan 13 at 19:50




          $begingroup$
          Does your strategy work if we restrict everything to the naturals?
          $endgroup$
          – Stupid Questions Inc
          Jan 13 at 19:50












          $begingroup$
          @StupidQuestionsInc Sure, if you can find the solution of the last equation with all $x_{i}>0$ and $y>1$.
          $endgroup$
          – Seewoo Lee
          Jan 13 at 22:06




          $begingroup$
          @StupidQuestionsInc Sure, if you can find the solution of the last equation with all $x_{i}>0$ and $y>1$.
          $endgroup$
          – Seewoo Lee
          Jan 13 at 22:06


















          draft saved

          draft discarded




















































          Thanks for contributing an answer to Mathematics Stack Exchange!


          • Please be sure to answer the question. Provide details and share your research!

          But avoid



          • Asking for help, clarification, or responding to other answers.

          • Making statements based on opinion; back them up with references or personal experience.


          Use MathJax to format equations. MathJax reference.


          To learn more, see our tips on writing great answers.




          draft saved


          draft discarded














          StackExchange.ready(
          function () {
          StackExchange.openid.initPostLogin('.new-post-login', 'https%3a%2f%2fmath.stackexchange.com%2fquestions%2f3068520%2fan-analogue-to-eulers-sum-of-powers-conjecture%23new-answer', 'question_page');
          }
          );

          Post as a guest















          Required, but never shown





















































          Required, but never shown














          Required, but never shown












          Required, but never shown







          Required, but never shown

































          Required, but never shown














          Required, but never shown












          Required, but never shown







          Required, but never shown







          Popular posts from this blog

          Can a sorcerer learn a 5th-level spell early by creating spell slots using the Font of Magic feature?

          Does disintegrating a polymorphed enemy still kill it after the 2018 errata?

          A Topological Invariant for $pi_3(U(n))$